next up previous contents
Nächste Seite: Ungleichung von Doob Aufwärts: Martingale Vorherige Seite: Submartingale und Supermartingale; Beispiele   Inhalt


Gleichgradige Integrierbarkeit

Sei $ T$ eine endliche Stoppzeit, und der stochastische Prozess $ \{X_t,\,t\ge 0\}$ sei càdlàg mit $ {\mathbb{E}\,}
\vert X_t\vert<\infty$ für jedes $ t\in\mathbb{R}$. In Korollar 3.2 hatten wir gezeigt, dass dann $ X_T$ eine wohldefinierte Zufallsvariable ist.

Zur Herleitung von Bedingungen, so dass auch $ {\mathbb{E}\,}\vert X_T\vert<\infty$ gilt, benötigen wir den Begriff der gleichgradigen Integrierbarkeit von Zufallsvariablen.

Hierfür benutzen wir die folgende Schreibweise: Sei $ X:\Omega\to\mathbb{R}$ eine beliebige Zufallsvariable mit $ {\mathbb{E}\,}\vert X\vert<\infty$. Für jedes $ A\in\mathcal{F}$ setzen wir dann $ {\mathbb{E}\,}[X;A]={\mathbb{E}\,}(X{1\hspace{-1mm}{\rm I}}(A))$.

Definition
$ \;$ Die Folge $ X_1,X_2,\ldots:\Omega:\to\mathbb{R}$ von Zufallsvariablen heißt gleichgradig integrierbar, wenn $ {\mathbb{E}\,}
\vert X_n\vert <\infty$ für jedes $ n\ge 1$ und

$\displaystyle \lim_{x\to\infty}\bigl(\sup_{n\ge 1} {\mathbb{E}\,}[\vert X_n\vert; \vert X_n\vert>x]\bigr)=0\,.$ (14)

Lemma 3.6   Die Folge $ \{X_n,\,n\ge 1\}$ ist genau dann gleichgradig integrierbar, wenn
(i)
$ \sup_{n\ge 1}{\mathbb{E}\,}\vert X_n\vert<\infty$ und
(ii)
wenn es für jedes $ \varepsilon>0$ ein $ \delta>0$ gibt, so dass $ {\mathbb{E}\,}[\vert X_n\vert;A]<\varepsilon$ für jedes $ n\ge 1$ und für jedes $ A\in\mathcal{F}$ mit $ P(A)<\delta$.

Beweis
$ \;$

Die Bedeutung der gleichgradigen Integrierbarkeit für die Konvergenz von Zufallsvariablen wird durch das folgende Lemma deutlich.

Lemma 3.7   Sei $ X_1,X_2,\ldots:\Omega\to\mathbb{R}$ eine beliebige Folge von Zufallsvariablen mit $ {\mathbb{E}\,}
\vert X_n\vert <\infty$ für jedes $ n\ge 1$ und $ \lim_{n\to\infty}X_n=X$ mit Wahrscheinlichkeit $ 1$ für eine gewisse Zufallsvariable $ X:\Omega\to\mathbb{R}$. Dann sind die beiden folgenden Aussagen äquivalent: (a) $ \{X_n,\,n\ge 1\}$ ist gleichgradig integrierbar. (b) Es gilt $ {\mathbb{E}\,}\vert X\vert<\infty$ und $ \lim_{n\to\infty}{\mathbb{E}\,}\vert X_n-X\vert=0$.

Beweis
$ \;$


Korollar 3.3   Sei $ X_1,X_2,\ldots:\Omega\to\mathbb{R}$ eine Folge von Zufallsvariablen, so dass $ {\mathbb{E}\,}
\vert X_n\vert <\infty$ für jedes $ n\ge 1$ und $ \lim_{n\to\infty}X_n=X$ mit Wahrscheinlichkeit $ 1$. Wenn $ \{X_n,n\ge 1\}$ gleichgradig integrierbar ist, dann gilt $ {\mathbb{E}\,}\vert X\vert<\infty$ und $ \lim_{n\to\infty}{\mathbb{E}\,}X_n={\mathbb{E}\,}X$.

Beweis
$ \;$ Wegen $ \vert{\mathbb{E}\,}X_n-{\mathbb{E}\,}X\vert\le{\mathbb{E}\,}\vert X_n-X\vert$ ergibt sich die Behauptung unmittelbar aus Lemma 3.7.

$ \Box$


next up previous contents
Nächste Seite: Ungleichung von Doob Aufwärts: Martingale Vorherige Seite: Submartingale und Supermartingale; Beispiele   Inhalt
Ursa Pantle 2005-07-13